Income Tax- Practice Exam

Lakukan tugas rumah & ujian kamu dengan baik sekarang menggunakan Quizwiz!

Colin, aged 12, has $11,650 of earned income from a newspaper route and $1,800 of unearned income. How much is taxable to Colin at Colin's tax rate? $400 $1,450 $12,000 $13,450

$1,450 Since Colin's unearned income is below $2,200 (in 2020), all of his income in excess of the standard deduction is taxed at his rate.His standard deduction is $11,650 + $350 = $12,000.His total income is $13,450 - $12,000 standard deduction = $1,450.

George owns a rental home in South Carolina. He decided that he would like to acquire a rental home in California. Will, who lives in California, has a rental home. For health purposes, Will must relocate to South Carolina and would like a rental home in that state. Will's basis is $125,000, and George's basis is $100,000. Will and George exchange the two properties under Section 1031 of the code, but George needs to give Will an additional $30,000 in cash. The fair market value of Will's property is $150,000, and the fair market value of George's property is $120,000. What is Will's recognized gain or loss? $0 $25,000 gain $30,000 gain $20,000 gain

B Will recognizes the lesser of the boot received ($30,000) or his realized gain ($25,000).

In 2020, Greg had a Section 1231 gain of $27,000. In the prior years, Greg had the following Section 1231 transactions: Year: Net Section 1231 Transaction: 2019: $6,000 Section 1231 Loss 2018: $7,000 Section 1231 Loss 2017: No Section 1231 Transaction 2016: $5,000 Section 1231 Gain 2015: No Section 1231 Transaction 2014: $10,000 Section 1231 Gain 2013: $2,000 Section 1231 Gain How will Greg's Section 1231 gain be taxed in 2020? $14,000 will be taxed as ordinary income. $27,000 will be taxed as a Section 1231 capital gain. $13,000 will be taxed as ordinary income, and $1,000 will be taxed as a Section 1231 capital gain. $13,000 will be taxed as ordinary income, and $14,000 will be taxed as a Section 1231 capital gain.

D Greg would have to recognize $13,000 of his 2020 gain as ordinary income since in 2019 and 2018 he had Section 1231 losses. The Section 1231 5-year look-back rule requires that a net Section 1231 gain in the current tax year will be taxed at ordinary income tax rates to the extent of any unrecaptured Section 1231 losses claimed during the last 5 years. The remaining $14,000 of Greg's Section 1231 gain (that is, $27,000 − $13,000) would be treated as a Section 1231 capital gain.

XYZ Inc. has the following net income before any application of net operating losses (NOLs): 2016: $9,000 Net income 2017: $12,000 Net income 2018: -$20,000 Net income 2019: $10,000 Net income 2020: $6,000 Net income What is the net income for 2020 after applying any NOL applicable? $1,200 $3,000 $4,800 $6,000

The correct answer is (A). After 2017, NOLs can only be carried forward and are limited to 80 percent of income. For 2018, 80 percent of $10,000 equals $8,000. For 2020, 80 percent of $6,000 equals $4,800, which leaves $1,200 in income. The NOL, after using it in 2020, equals $7,200, or $20,000 − $8,000 − $4,800.

A taxpayer who is an employee earning $150,000 is responsible for paying how much in the form of Social Security and Medicare taxes in 2020? $10,712 $22,950 $11,475 $9,300

The correct answer is (A) The Social Security wage base for 2020 is $137,700.6.2% × $137,700 = $8,537There is no wage base for Medicare FICA taxes, so all $150,000 is taxed at 1.45 percent to the employee.$150,000 × 1.45% = $2,175$8,537 + $2,175 = $10,712

Arnold and Phoebe have been married for 20 years and always file a joint return, but they never itemize their deductions. They have a gross income of $80,000 and deductions for adjusted gross income (AGI) in the amount of $5,000, but they do not have any children. Neither Arnold nor Phoebe are over the age of 65, and neither is blind. What is Arnold and Phoebe's taxable income in 2020? $50,200 $55,200 $62,600 $75,000

The correct answer is (A). Arnold and Phoebe's taxable income is equal to their gross income less deductions for adjusted gross income less the greater of the standard deduction or itemized deductions.

Ian's valuable gun collection was stolen from his home. He purchased the guns for $100,000, and they were worth $125,000 at the time they were stolen. He did not list them separately on his homeowners insurance policy, so he only received $1,000 from insurance. How much may he deduct as a casualty loss on his tax return if his adjusted gross income (AGI) is $100,000? $0 $83,900 $88,900 $99,000

The correct answer is (A). Casualty losses that are not declared national disasters are no longer deductible (2017 Tax Cuts and Jobs Act).

Jordan has a real estate asset used in his business. He exchanges it for a like-kind real estate asset owned by Charles, which is worth $36,000. The basis of Jordan's asset is $40,000, and he gives Charles $20,000 cash plus the asset in exchange for Charles's asset. Charles's basis in his original asset is $10,000. What is Charles's gain or loss? $20,000 gain recognized $26,000 gain realized and recognized $0 gain recognized $0 loss recognized

The correct answer is (A). Charles must recognize gain to the extent of boot paid to him, that is, $20,000. The values exchanged must be equal. Jordan exchanges his property plus $20,000 for a property valued at $36,000; therefore Jordan's asset must be worth $16,000. Charles receives a boot of $20,000.Jordan adds a boot of $20,000 to his old basis so that the value of what is exchanged equals the new basis. Charles receives a boot of $20,000.The recognized gain equals the boot of $20,000 that is received.

Two years ago, Chloe purchased 100 shares of Pacific Journey, Inc., for $15,000. Unfortunately, the value of the shares has dropped to $10,000. Chloe's daughter, Emma, is heading off to college, and Chloe is tired of waiting for a return on the stock. Chloe gives the stock to Emma when it is worth $10,000 to help fund Emma's education. If Emma sells the shares 3 months after the transfer for $12,000, what is the amount and character of her gain or loss? $0 $2,000 short-term capital gain $3,000 long-term capital loss $3,000 short-term capital loss

The correct answer is (A). Chloe gave loss property to her daughter. When a taxpayer transfers property with a loss to someone else and the recipient sells the property for an amount between the donor's adjusted basis and the fair market value of the stock on the date of the gift, no gain or loss is recognized. In this example, the no gain/no loss corridor is from $10,000 (the fair market value of the stock on the date of the gift) to $15,000 (the adjusted basis in the hands of the donor). If Emma sells the stock for any amount between $10,000 and $15,000, she is not required to recognize any gain and she is prohibited from recognizing any loss on the transaction. Since there is no gain or loss, there is no need to categorize the tax result as either long-term or short-term.

Dorian, single and aged 42, has the following items of income and expense for the current tax year: • Wages: $60,000 • Interest: $1,000 • Inheritance: $50,000 • Alimony paid: $10,000 (2016 divorce decree) • Child support paid: $8,000 • Federal taxes paid: $5,000 • State income taxes paid: $2,000 • Medical expenses: $7,500 How much of the medical expenses will he deduct on Schedule A? $0 $2,400 $5,100 $7,500

The correct answer is (A). He could deduct the excess of medical expenses over the 7.5 percent hurdle of adjusted gross income (AGI). This would result in his needing to pay only $3,675 of the medical expenses.$51,000 × .075 = $3,825$7,500 − $3,825 = $3,675However, Dorian will not do so because his standard deduction of $12,400 exceeds the sum of $3,675 (medical) and $2,000 (state income taxes paid). Therefore he will use the standard deduction.

Frank and Gina are trying to calculate their gross income. Which of the following items should they exclude from their gross income? I. $75,000 in cash inherited by Gina from her mother II. $30,000 borrowed by Frank and Gina from First City Bank III. A $10,000 gain from the sale of Frank and Gina's boat IV. $600 of interest earned on a loan made by Frank to his cousin Michael I and II III and IV I, II, and III I, II, and IV

The correct answer is (A). Inheritance is not income. Inherited cash or property is excluded from gross income, so Option I is correct. Borrowed money is also excluded from gross income, so Option II is also correct. Gain on the sale of assets (Option III) and interest income (Option IV) are both included in gross income.

Rylee gave her grandson, Kim, stock worth $500,000 this year, in addition to a previous gift equal to the annual gift tax exclusion amount. She purchased the stock for $250,000 several years earlier and believed that the value would increase substantially in the near future. Since she had already used up her lifetime gift tax exemption in prior tax years, Rylee paid $200,000 in gift taxes on the transfer. If Kim sells the stock 6 months after the transfer for $750,000, which of the following statements is correct? Kim will realize a $400,000 long-term capital gain. Kim will realize a $250,000 long-term capital gain. Kim will realize a $500,000 long-term capital gain. Kim will realize a $250,000 short-term capital gain.

The correct answer is (A). Kim received the stock by gift, so he qualifies for a carryover basis. Rylee's original basis in the property was $250,000. Since Rylee paid gift tax on the transfer, Kim is permitted to increase his basis by the portion of the gift tax paid that represents gain. The portion of the gift that represents gain is 50 percent (that is, $250,000 appreciation in the property divided by $500,000 fair market value of the property as of the date of gift). Fifty percent of the gift taxes paid equals $100,000. Kim's basis is therefore $350,000. If he sells the stock 6 months after the transfer for $750,000, he will realize a $400,000 gain. Because Rylee transferred appreciated property to Kim, her holding period is added to Kim's holding period for the asset, transforming the gain into a long-term capital gain.

Morgan receives nonqualified stock options (NQSOs) with an exercise price of $50 when the stock is trading at $50. Morgan exercises these options 2 years after the date of the grant when the stock price is $65 per share. Which of the following statements is correct? Morgan will have W-2 income of $15 per share upon exercise. Morgan will have W-2 income of $65 per share upon exercise. Morgan will have $50 of alternative minimum taxable income (AMTI) upon exercise. Morgan's adjusted basis for regular income tax will be $15 at exercise.

The correct answer is (A). Morgan will have W-2 income of $15 per share upon exercise.$65 − $50 = $15

Julio is starting a new business. His major concern is limiting his legal liability, but he is also concerned with the ability of the company to raise capital. At some point, he would like to be able to easily sell partial interests in the business to other investors or perhaps even take the company public, so he does not want to limit the number of potential owners. Julio will be actively working in the business and would like to receive the same tax-advantaged fringe benefits as other employees. Which of the following entities would best suit Julio's needs? A C corporation An S corporation An LLC A general partnership

The correct answer is (A). Option (B) is incorrect because Julio does not want to limit the number of potential owners and an S corporation cannot have more than 100 owners. Option (C) is incorrect; if Julio is interested in taking the company public in the future, the LLC form would not be appropriate. Option (D) is incorrect because a partnership would not provide the limited liability in which Julio is interested. LLC members, partners, and greater-than-2-percent S corporation owners are not eligible to receive the same tax-advantaged fringe benefits as other employees. (They are taxed on the benefits that they receive.)

Five years ago, Edmund purchased 200 shares of Rainbow Books, Inc., for $4,000. Edmund recently gifted those shares of stock to his son, Peter. The value of the 200 shares of stock on the date of the gift was $2,000. Which of the following statements is correct? If Peter subsequently sells the shares of Rainbow Books, Inc., for $4,750, the basis used to calculate his gain or loss will be $4,000. If Peter subsequently sells the shares of Rainbow Books, Inc., for $4,750, the basis used to calculate his gain or loss will be $1,000. If Peter subsequently sells the shares of Rainbow Books, Inc., for $1,600, the basis used to calculate his gain or loss will be $4,000. If Peter subsequently sells the shares of Rainbow Books, Inc., for $1,600, he will not have any gain or loss.

The correct answer is (A). Peter will have a double basis in the stock, determined as follows: If the donee's sale price is... less than full market value... then donees basis is: the loss basis between the original basis and the full market value... no gain or loss greater than the original basis... The gain basis If Peter subsequently sells the shares of Rainbow Books, Inc., for $4,750 (or greater than the full market value on the date of the gift), the basis used to calculate his gain or loss will be $4,000, the gain basis.

On January 1 of this year, Patti purchased a home worth $1.5 million with an interest-only mortgage of $1.2 million. She is currently only paying interest on the mortgage at the rate of 5 percent annually. What amount of qualified residence interest may she deduct as home mortgage interest on Schedule A of her individual income tax return this year? $37,500 $50,000 $55,000 $60,000

The correct answer is (A). Since Patti purchased her home after December 15, 2017, the deduction for qualified residence interest is limited to the interest on $750,000 of principal. The calculation is simply the interest rate (5 percent) multiplied by the qualifying limit of $750,000; this equals $37,500.

Which of the following is NOT a requirement of the individual real estate investor exception to the passive-activity loss rules? The taxpayer must materially participate in the activity. The taxpayer must own at least 10 percent of the value of the real estate. The taxpayer must have an adjusted gross income (AGI) of less than $150,000. The taxpayer must actively participate in the activity.

The correct answer is (A). The taxpayer is not required to materially participate in the activity, but the taxpayer must actively participate in the activity. Material participation requires substantial, continuous involvement in the operation of the activity. Active participation means that the taxpayer participates in making management decisions concerning the property but is not substantially and continuously involved in the operation of the activity.

What is the usual federal income tax treatment of the employer's premium payments under a group long-term disability income insurance plan? A)The amount of the premium payments is excluded from the employee's income, regardless of whether or not the plan is discriminatory. B)The amount of the premium payments is fully included in the employee's income if the plan is discriminatory. C)The amount of the premium payments is included in the employee's income to the extent the plan will replace more than 50 percent of an employee's earnings. D)The amount of the premium payments is excluded from the employee's income only if the plan is nondiscriminatory.

The correct answer is (A). When group long-term disability insurance is paid for by the employer, the amount of the premium payments is excluded from the employee's income even if the plan is discriminatory.

Frank has been adjusting his investment portfolio to meet his target asset allocation and has realized the following capital gains and losses this year: • $13,000 in short-term capital gains • $9,000 in short-term capital losses • $4,000 in long-term capital gains • $7,000 in long-term capital losses What is Frank's net capital gain or loss? A $1,000 net short-term capital gain (NSTCG) A $1,000 net long-term capital loss (NLTCL) A $1,000 net short-term capital loss (NSTCL) A $1,000 net long-term capital gain (NLTCG)

The correct answer is (A).Frank would have a $1,000 net short-term capital gain (NSTCG). Short Term: Gains: $13,000 Losses: $9,000 NSTCG: $4,000 Long Term: Gains: $4,000 Losses: $7,000 NLTCL: $3,000 NSTCG: $4,000 NSTCL: $3,000 NSTCG: $1,000

Two years ago, Diane purchased 100 shares of the Colbert Corporation for $15,000. On January 1 of this year, Diane reviews her investment portfolio and finds out that she has had a very profitable year. To offset some of her gains, Diane sells her shares of the Colbert Corporation for $10,000. On January 25 of the same year, Diane reads a newspaper article indicating that the price of the Colbert Corporation is expected to increase substantially. Second-guessing the wisdom of selling her previous shares of Colbert stock, she purchases 100 shares of the Colbert Corporation for $8,000. What are the tax consequences for Diane this year? Diane will have a $5,000 realized, but not recognized, loss. Diane will have an $8,000 realized and recognized loss. Diane will have a $5,000 realized and recognized loss. Diane will have a $7,000 realized, but not recognized, loss.

The correct answer is (A).Since Diane purchased and sold substantially identical securities within 30 days, a wash sale occurs. Her realized loss on the sale of the original shares is calculated as follows: Amount Realized: $10,000 Less: Adjusted Basis: − $15,000 Equals: Gain or Loss: -$5,000 Due to the wash sale transaction, however, Diane will not be permitted to recognize the loss in the year it was incurred. Instead, the realized but unrecognized loss of $5,000 will be added to the basis of the replacement securities.

Destiny owns a downtown office building. Destiny originally purchased the building for $900,000, and she took depreciation deductions of $400,000. The straight-line depreciation would have been $300,000. What would be the tax consequences if Destiny sold the building for $2,100,000? Destiny would have ordinary income of $400,000. Destiny would have $300,000 of unrecaptured Section 1250 gain. Destiny would have capital gains of $1,600,000. None of the above would occur.

The correct answer is (B). Sale Price: $2,100,000 Less Adjusted Basis (that is, $900,000 − $400,000): − $500,000 Equals Gain: $1,600,000 Tax Treatment of Gain: Amount of Gain $100,000 $300,000 $1,200,000 Treatment of Gain Ordinary income 25 percent tax rate Capital gain Explanation of Calculation Excess depreciation (Total depreciation less straight-line depreciation) Straight-line depreciation Excess gain under Section 1231

XYZ Inc. has the following net income before any application of net operating losses (NOLs): 2016: $9,000 Net income 2017: $12,000 Net income 2018: -$20,000 Net income 2019: $10,000 Net income 2020: $6,000 Net income What is the income for 2019 after applying any applicable NOL? $0 $2,000 $10,000 $12,000

The correct answer is (B). After 2017, NOLs can only be carried forward and are limited to 80 percent of income. 80% × $10,000 = $8,000$10,000 − $8,000 = $2,000

In 2020, Lamark, a single-filing taxpayer, has an alternative minimum taxable income (AMTI) of $558,400. What is Lamark's alternative minimum tax (AMT) exemption this year? $40,000 $62,900 $72,900 $10,000

The correct answer is (B). Because Lamark's AMTI is above the AMT phaseout threshold amount, his AMT exemption must be reduced. Lamark's exemption is reduced by 25 percent of the amount that his AMTI exceeds $518,400 (the threshold). Therefore Lamark's exemption must be reduced by $10,000, that is, ($558,400 − $518,400) × 0.25. As a result, Lamark is entitled to an exemption of $62,900, that is, $72,900 − $10,000.

John is a CPA, and Jack operates a laundromat. Instead of paying for each other's services in cash, they decide to exchange accounting services for laundromat services. The value of John's accounting service is $750, and the value of Jack's laundromat service is $600. Which of the following statements is correct? John must include $750 in his gross income. John must include $600 in his gross income. Jack must include $600 in his gross income. Neither must include any amount in gross income since barter transactions are tax-free.

The correct answer is (B). John must include $600 (the value of consideration received) in his gross income, and Jack must include $750 (the value of consideration received) in his gross income.

Katie is a 15 percent owner in Enchanted Creations, LLC, a very successful web-developing business. She is also a 15 percent owner in Stronghold, LLC, an internet-based virus protection service. She materially participates in Enchanted Creations but does not materially participate in Stronghold. She has an at-risk amount of $600,000 and an income of $250,000 for Enchanted Creations. For Stronghold, she has an at-risk amount of $75,000 and a loss of $200,000. What amount of loss is suspended because of the passive-activity income rules? $0 $75,000 $125,000 $200,000

The correct answer is (B). Katie will not be allowed to take any loss for the current year. Of the $200,000 loss from Stronghold, $125,000 will be suspended because of the at-risk rules (she only has $75,000 at risk) and $75,000 will be suspended because of the passive-activity loss rules. (She does not have any passive income to offset the loss since she materially participates in Enchanted Creations.)

Liam paid $50,000 for a 30 percent interest in a general partnership in which he materially participates. The partnership loss for the year is $180,000. How much can Liam deduct? $0 $50,000 $54,000 $60,000

The correct answer is (B). Liam's loss is limited to $50,000 due to the at-risk rules. He will also have a suspended loss of $4,000, that is ($180,000 × 30%) − $50,000.

Marcus has a 30 percent interest in a general partnership for which he paid $25,000. The partnership loss for the year is $180,000. How much can Marcus deduct? $0 $25,000 $54,000 $60,000

The correct answer is (B). Marcus's loss is limited to $25,000 due to the at-risk rules. He will also have a suspended loss of $29,000, that is, ($180,000 × 30%) − $25,000.

Randy is starting a new business. He is concerned about legal liability. He would like to have flow-through taxation. At some point, he would like to be able to easily sell interests in the business, but he does not expect to have more than 20 investors. Randy does not want to pay self-employment taxes on all income. Which of the following entities would best suit Randy's needs? A proprietorship An S corporation A C corporation A partnership

The correct answer is (B). Option (A) is incorrect because a proprietorship does not provide limited liability and Randy would need to change entities to take on future investors. Option (C) is incorrect because a C corporation would not provide flow-through taxation. Option (D) is incorrect because a partnership would not provide limited liability and Randy would need to pay self-employment taxes on the business net income. Only option (B), an S corporation, meets all of Randy's requirements.

OPQ, Inc., has a net income of $650,000 before depreciation and has purchased one used piece of equipment during the year for $1,200,000. What Section 179 depreciation is allowable? $0 $650,000 $1,000,000 $1,200,000

The correct answer is (B). Section 179 is limited to the lesser of $1,040,000 (for 2020) and the net income of $650,000.

All of the following statements concerning the alternative minimum tax (AMT) as it applies to individual taxpayers are correct EXCEPT: Some itemized deductions taken for regular tax purposes must be added back to income when calculating the AMT. Taxpayers are permitted to take the standard deduction for both regular and AMT tax purposes. All adjustments made to itemized deductions when calculating AMT result in a permanent increase in tax. Charitable deductions may be claimed in the same manner for regular and AMT tax purposes.

The correct answer is (B). Taxpayers who do not itemize deductions take the standard deduction for regular tax purposes, but this is added back to alternative minimum taxable income (AMTI) for AMT purposes. All of the other statements are correct.

The alternative minimum tax (AMT) was originally designed to: create a more user-friendly tax system. curb abuses by high-income taxpayers. provide additional credits to certain low-income taxpayers. give taxpayers a choice of which tax to pay.

The correct answer is (B). The AMT was enacted in 1986 to curb perceived abuses by high-income taxpayers who were trying to minimize their current income tax liability.

XYZ, Inc., has a net income of $2,650,000 before depreciation and has purchased two pieces of equipment during the year. The first piece of equipment cost $1,900,000, and the second piece of equipment cost $800,000. How much may be expensed under Section 179 for the current year for both pieces of equipment? $110,000 $930,000 $1,040,000 $2,700,000

The correct answer is (B). The Sec. 179 expensing maximum is $1,040,000 (for 2020). This amount is reduced dollar-for-dollar by the amount of depreciable property placed in service during the year exceeding $2,590,000 (for 2020). The total placed in service was $2,700,000, which exceeds $2,590,000 by $110,000. The reduced limit is therefore $1,040,000 − $110,000 = $930,000.

Gina spent $5,000 in day-care services for her three children so that she could work. Assuming her earned and adjusted gross income is $100,000, how much is her dependent care credit? $1,200 $1,000 $5,000 $3,000

The correct answer is (B). The dependent care credit is not phased out, and it provides a credit of 20 percent on up to $3,000 per qualifying child with a maximum of $6,000 for two or more children. However, since Gina's actual expenses are limited to $5,000, her credit is calculated as $5,000 × 0.20 = $1,000.

Yolanda became an alternative minimum tax (AMT) taxpayer last year. She had to add several items to her regular taxable income to arrive at her alternative minimum taxable income (AMTI). Which of the following items will result in an AMT credit that can be used to offset future regular tax liability? $10,000 in state income taxes A $90,000 difference between the fair market value of stock and the strike price in the incentive stock option (ISO) used to purchase the stock $6,000 in interest on private activity municipal bonds $6,000 in home equity loan interest

The correct answer is (B). The inclusion of the difference between the fair market value and exercise price of the stock options will result in a credit that Yolanda can use against future regular income tax liability. Options (A) and (C) are adjustments, which result in permanent differences in tax liability as a result of the imposition of the AMT. Home equity loan interest is not deductible for regular tax or AMT.

Contributions to charity are limited to a certain percentage of income. How long is the carry-over period for individuals to use any excess current charitable deductions? 1 year 5 years 7 years 15 years

The correct answer is (B). There is a 5-year carry-over provision for charitable deductions.

Kim raises and sells horses as a hobby. She also has income from riding lessons that are part of the hobby activity. Income from Horse Sales:$10,000 Income from Riding Lessons:$5,000 Feeding Expenses:$8,000 Boarding Expenses: $6,000 Veterinary Expenses: $2,000 How much of the income is taxable? $5,000 $15,000 $0 -$1,000

The correct answer is (B). This is a hobby activity. All income is taxable.

Which of the following would be added to a taxpayer's regular taxable income to arrive at his or her alternative minimum taxable income (AMTI)? Receipt of interest on public-purpose municipal bonds Receipt of interest on private activity municipal bonds Exercise of nonqualified stock options Sale of the shares purchased through the exercise of incentive stock options (ISOs)

The correct answer is (B). While private activity municipal bonds generate interest that is exempt for regular income tax purposes, once a taxpayer is subject to the alternative minimum tax (AMT), the interest generated by these bonds is taxable and must be added to regular taxable income to arrive at AMTI. The interest earned on public-purpose municipal bonds is always exempt from tax under the regular or AMT system. The exercise of a nonqualified stock option will generate ordinary income that is taxed for regular income tax purposes but would not need to be added back to calculate AMTI. The sale of shares purchased through the exercise of ISOs generates a negative adjustment to AMTI.

Clark invested $100,000 in an annuity contract many years ago. This year, Clark annuitized the contract. The insurance company agreed to pay Clark $520.83 per month for 20 years. Assuming that Clark receives eight payments this year, how much can Clark exclude from his gross income this year? $833.33 $3,333.31 $4,164.64 $6,249.96

The correct answer is (B). $3,333.31 Clark's expected return is $125,000, that is, 20 years × 12 months × $520.83. Therefore his exclusion ratio is 80 percent, or $100,000 ÷ $125,000. Clark will receive $4,166.64 in annuity payments this year (8 payments × $520.83), of which $3,333.31 can be excluded. Therefore Clark must include $833.33, or 20% × $520.83 × 8, in his income this year.

Eleanor loans $30,000 to her son James and does not charge any interest. James has investment income of $2,500 and investment expenses of $500. Assume that the applicable federal rate (AFR) is 3 percent. How much interest must be imputed on the loan? $2,500 $900 $2,000 $1,000

The correct answer is (B). $900 James has net investment income of $2,000. Therefore the amount of imputed interest is the lesser of net investment income and the AFR-calculated interest minus the interest based on the stated rate of the loan. Since the stated rate of interest on the loan is 0 percent, the amount of imputed interest is the lesser of $2,000 and $900 (that is, $30,000 × 0.03). Therefore $900 of interest must be imputed on the loan.

During her working years, Brianna ran an antiques store that was very successful in the local community. When she retired, she kept some of the display cases in order to display her own collections in her home. The display cases were originally purchased for the business at a cost of $15,000 and were fully depreciated by the time Brianna retired. Brianna died last month, and the display cases were valued in her estate at $8,000. If Brianna's daughter, Ingrid, inherits the display cases and sells them 2 months after Brianna's death for $8,500 , what will be the income tax treatment on the sale? $500 ordinary income $500 long-term capital gain $500 short-term capital gain $8,000 ordinary income

The correct answer is (B).Even though the display cases were Section 1231 assets in Brianna's hands and were subject to depreciation recapture, once they ran through Brianna's estate, they qualified for a Section 1014 step to fair market value in basis, which eliminates the recapture potential. If Ingrid sells the cases for $8,500 2 months after Brianna's death, her gain will be $500 due to the step up in basis. Since all assets passing through an estate and receiving a step up in basis qualify for long-term capital-gains treatment, the gain will be characterized as a long-term gain.

The Tudor Company grants Edward one incentive stock option (ISO) on January 10, 2019. The exercise price is $10. The market price on the exercise date (June 12, 2020) is $33. What is the alternative minimum tax (AMT) consequence when Edward exercises the ISO? $0 AMT gain $10 AMT gain $23 AMT gain $33 AMT gain

The correct answer is (C). The AMT gain is the difference between the market price and the exercise price at the date of exercise.$33 − $10 = $23

Joshua has three capital transactions during the current year: • A short-term capital loss (STCL) of $5,000 • A short-term capital gain (STCG) of $3,000 • A long-term capital loss (LTCL) of $2,000 What is the net effect on Joshua's income taxes if he is in the 35 percent income tax bracket? A $1,400 tax reduction A $1,050 tax reduction An $850 tax reduction A $450 tax reduction

The correct answer is (B).The STCG and the STCL can be netted: Short-Term Gains: $3,000 Losses: + -$5,000 NSTCL: -$2,000 Netting the $2,000 LTCL and the $2,000 STCL gives Joshua a total loss of $4,000. Joshua can only use $3,000 of losses to offset his ordinary income at 35 percent: $3,000 × 0.35 = $1,050 The remaining $1,000 is a long-term capital-loss carryover.

Jason owns a gym that specializes in training competitive athletes. Several years ago, Jason purchased a new weight machine for $2,000 and has since taken $600 in depreciation deductions. Jason is now ready to replace the weight machine with a more current model, but he does not know how to calculate the tax consequences of selling the old weight machine. Which of the following statements is true regarding the tax consequences of selling the old weight machine? If Jason sells the old weight machine for $1,400, he will have a $600 capital loss. If Jason sells the old weight machine for $1,200, he will have a $600 ordinary gain. If Jason sells the old weight machine for $1,600, he will have an ordinary gain of $200. If Jason sells the old weight machine for $2,200, he will have an $800 capital gain.

The correct answer is (C).

All of the following are requirements for the deferral of gain in a nonsimultaneous exchange under Section 1031 EXCEPT: The replacement property must be like-kind property with respect to the original property. The proceeds from the sale of the original property must be held by an escrow agent. A replacement property must be identified within 90 days of the sale of the original property. The closing on the replacement property must take place by the earlier of 180 days from the sale of the original property and the due date (including extensions) of the tax return for the year in which the original property was sold.

The correct answer is (C). A replacement property must be identified within 45 days, not 90 days, of the sale of the original property.

Kurt, Adrian, and Leonard each have an ownership interest in Three Guys' Hot Dogs, Inc. Based on the following information, which of them is (are) considered to have materially participated in the conduct of the Three Guys' Hot Dogs business this year? I. Kurt dedicated more than 500 hours this year to Three Guys' Hot Dogs. II. Adrian devoted 150 hours to Three Guys' Hot Dogs this year. III. Leonard devoted 115 hours to Three Guys' Hot Dogs this year, but he also devoted more than 100 hours to several other activities for a total of 520 hours in all of the activities combined. I only II and III I and III I, II, and III

The correct answer is (C). Adrian has not materially participated. Although Adrian devoted more than 100 hours to the activity, he did not devote more hours than anyone else because Kurt worked at Three Guys' Hot Dogs for more than 500 hours. Leonard is also a material participant because he devoted more than 100 hours to the activity and also devoted more than 100 hours to several other activities for a total of more than 500 hours in all of the activities combined.

Gabriel and Anna got married and bought a house 15 months ago. Anna's job recently transferred her to an office in a different state, so Gabriel and Anna sold their house. What is the maximum amount of gain from the sale of the personal residence that Gabriel and Anna can exclude from their gross income? $0 $250,000 $312,500 $500,000

The correct answer is (C). Although they did not live in their house for a full 2 years, Gabriel and Anna are eligible for a prorated exclusion because of Anna's change in employment. Therefore they are eligible for a maximum exclusion of $312,500, that is, ¹⁵⁄₂₄ × $500,000.

Gavin, an unmarried individual who is an executive at IT Consulting, Inc., was granted 1,500 ISOs on IT's stock 2 years ago when the price per share was $25. The last few years have resulted in tremendous growth for IT Consulting, and the stock is now trading at $55 per share. Gavin exercised the ISOs but did not sell the stock; he plans on holding the shares for at least a year so he can pay the lower capital-gains tax rate on the growth. How much will Gavin need to add to his taxable income when computing alternative minimum taxable income (AMTI) as a result of this transaction? $0 $37,500 $45,000 $82,500

The correct answer is (C). Even though exercise of the options results in no taxable event for regular tax purposes this year, Gavin will need to add $45,000, that is, ($55 − $25) × 1,500, to his taxable income when computing AMTI. If there are no other transactions this year that could reduce AMTI, it is likely that Gavin will become an alternative minimum tax (AMT) taxpayer for the year since the tax preference item—the gain on the exercise of the ISO—is greater than his exemption for AMT purposes.

Bryan and Diane are trying to calculate their gross income. Which of the following items should they exclude from their gross income? I. A $25,000 gift from Diane's mother for the down payment of their new house II. $30,000 borrowed by Bryan and Diane from First City Bank III. A $10,000 increase in the value of Delta stock, which they own in their brokerage account IV. $55,000-worth of home repair work that was exchanged for tax work by Bryan

The correct answer is (C). Gifts are not income. A gift of cash or property is excluded from gross income. Borrowed money is also excluded from gross income. Gain on assets not sold is not taxable until the assets are sold. However, barter transactions are taxable.

Which of the following is not a deduction for adjusted gross income (AGI)? Maintenance expenses for a rental property actively managed by the taxpayer Moving expenses of a taxpayer who is an active duty member of the Armed Forces Real estate taxes One-half of self-employment tax paid

The correct answer is (C). Real estate taxes are deductible from AGI and are limited to $10,000 after 2017. Moving expenses are a deduction for AGI for members of the Armed Forces. Self-employed taxpayers can receive a deduction for one-half of self-employment taxes paid, and maintenance expenses for an actively managed rental property are deducted for AGI.

Wanda, an unmarried mother, has four children, aged 5, 9, 13, and 18. What is the maximum amount of her Child tax credit for the current tax year assuming that she is under the adjusted gross income (AGI) threshold? $2,000 $4,000 $6,000 $8,000

The correct answer is (C). The 18-year-old does not qualify since the cutoff for children is under, but not including, age 17. Therefore Wanda will get a $2,000 tax credit for each of her three children, which will come to a total of $6,000.

Tyrone owns a cottage that he sometimes rents out to other people. Which of the following statements regarding Tyrone's rental activity is correct? If Tyrone rents out the cottage for less than half of the year, it will be considered a nontaxable activity. If Tyrone rents out the home for exactly half the year and uses the home personally for exactly half the year, the activity will be considered primarily a rental activity. If Tyrone rents out the home for 180 days per year and uses the home personally for 20 days of the year, the activity will be considered a mixed-use activity. If Tyrone rents out the home for more than half of the year, it will be considered primarily a rental activity regardless of his personal use.

The correct answer is (C). If Tyrone rents out the home for 180 days per year and uses the home personally for 20 days of the year, the activity will be considered a mixed-use activity. Option (A) is not correct because the activity will only be considered a nontaxable activity if Tyrone rents out the cottage for less than 15 days per year. Option (B) is not correct because this option meets the requirements of a mixed-use activity. That is, the property is rented out for 15 days or more, but the owner personally uses it for the greater of 14 days or 10 percent of the rental days. Option (D) is not correct because even if Tyrone rents out the home for more than half of the year, the activity may be a mixed-use activity if he personally uses the property for the greater of 14 days and 10 percent of the rental days.

John is a 30 percent partner in a partnership that produced business income (before adjustments) of $250,000 this year, and he is a 100 percent owner in an S corporation that produced net business income of $40,000 this year. The total amount of self-employment taxes allocated to his net business income is $5,299. His total taxable income is $110,000. What are John's qualified business income (QBI) and QBI deduction, respectively? $115,000 and $23,000 $115,000 and $22,000 $109,701 and $21,940 $110,000 and $22,000

The correct answer is (C). John's QBI from the partnership is $69,701, that is, 30% × $250,000 − $5,299, and his QBI from the S corporation is $40,000. His total QBI is $109,701, that is, $69,701 + $40,000. His QBI deduction is $21,940, that is, 20% × $109,701.

Karen and Jim, both aged 35, are married and filed a joint federal income tax return for 2020. Karen earned a salary of $128,000 and was covered by her employer's pension plan. Jim was not employed, and the couple had no other income. On June 15, Karen contributed $6,000 to an IRA for herself and $6,000 to an IRA for Jim. The allowable IRA deduction on Karen and Jim's 2020 joint tax return is $0. $3,000. $6,000. $12,000.

The correct answer is (C). Karen's IRA is not deductible due to the regulations concerning the phaseout of deductibility of a taxpayer who is an active participant. The phaseout for married taxpayers filing jointly begins at $104,000 and ends at $124,000 for 2020. Jim's IRA is fully deductible. Active participation in an employer's retirement plan by one spouse does not affect the full deductibility of the other spouse's IRA as long as the other spouse is not an active participant in an employer-sponsored retirement plan. Therefore the correct answer is $6,000, or Jim's contribution.

Sue is a 10 percent owner in Doggy Companion, LLC, a day-care center for dogs. She is also a 15 percent owner in Fashionista, LLC, a successful children's clothing store. She does not materially participate in either business. Her at-risk amount for Doggy Companion is $140,000, and she has a loss of $300,000 for the day care. For Fashionista, she has an at-risk amount of $50,000 and an income of $130,000. She also has wage income of $80,000 and capital-gain income of $30,000. Which of the following statements is true? The loss suspended because of the at-risk rules is $110,000, and the loss suspended because of the passive-activity loss rules is $60,000. The loss suspended because of the at-risk rules is $110,000, and the loss suspended because of the passive-activity loss rules is $0. The loss suspended because of the at-risk rules is $160,000, and the loss suspended because of the passive-activity loss rules is $10,000. The loss suspended because of the at-risk rules is $160,000, and the loss suspended because of the passive-activity loss rules is $80,000.

The correct answer is (C). Of the $300,000 loss from Doggy Companion, $160,000 will be suspended because of the at-risk rules (she only has $140,000 at risk) and $10,000 will be suspended because of the passive-activity loss rules. (She only has $130,000 of passive income.)

A well-known sculptor donated one of her original bronze creations to a local charity, which auctioned the piece for $3,000. The artist's costs were as follows: • Bronze: $425 • Other materials: $150 • Pro rata overhead: $125 • Furnace/casting fees: $200 • Artistic contribution: $2,100 Assuming this is the only charitable contribution and based on an annual income of $150,000, what is the maximum amount of charitable income tax deduction available to the artist? $3,000 $2,100 $775 $900

The correct answer is (C). Only materials and expenses are deductible, not artistic contribution or time. No deduction is allowed for use of property; therefore the pro rata overhead would likely not be allowed.

Payments for employment-related care that are made to relatives of the taxpayer may qualify for the credit for child- and dependent-care expenses. Which of the following payments does not qualify? Payments for employment-related care made to the taxpayer's aunt Payments for employment-related care made to the taxpayer's 21-year-old daughter (who is not a dependent of the taxpayer) Payments for employment-related care made to a dependent of the taxpayer Payments for employment-related care made to the taxpayer's 17-year-old niece

The correct answer is (C). Payments for employment-related care made to a dependent of the taxpayer do not qualify for the credit for child- and dependent-care expenses. All of the other options are qualifying payments.

Ralph is not married and does not have any children. However, Ralph is a very good son and provides more than half of the cost of maintaining a very nice apartment for his mother and more than half of her support since her only income is a small amount from Social Security. Which of the following filing statuses should Ralph use, and why? A)Single, because Ralph is not married B)Single, because Ralph does not have any qualifying children C)Head of household, because Ralph's mother is his dependent D)Head of household, because Ralph's mother is a qualifying child

The correct answer is (C). Ralph provides more than half of his mother's support and her income is below the exemption reference limit (which does not count Social Security income), so he can claim his mother as a dependent and he is eligible to use the head-of-household filing status. Option (D) is incorrect because Ralph's mother is a qualifying relative, not a qualifying child. Options (A) and (B) are incorrect; since Ralph is eligible to use the head-of-household filing status, he should use that filing status rather than the less advantageous single filing status.

Petra, a married-filing-jointly taxpayer, paid $10,000 of qualified tuition and related expenses for each of her twin sons, Jamie and Graham, during 2020. They started their freshman year of college during 2020. Petra was very excited that both sons excelled in the college environment, especially since Jamie had a drug addiction during his senior year of high school. Petra had a friend on the college admissions board who thankfully overlooked Jamie's felony drug conviction. Petra also paid $2,000 of qualified tuition and related expenses for her son Tom's sophomore year of college and $3,000 for her own master's degree program. Petra claims all three of her sons as dependents. Her modified gross income for the year is $50,000. What is the available American Opportunity tax credit for 2020? $2,000 $2,500 $4,500 $7,500

The correct answer is (C). The credit is not available for Jamie since he has a felony drug conviction. The maximumAmerican Opportunity tax credit of $2,500 is available to Graham. The credit available for Tom is $2,000, or $2,000 x 100%. Tom would have needed $4,000 of expenses to get the maximum credit. The total of credits for Graham and Tom is $4,500 for 2020.

Grace is single, and her share of qualified business income (QBI) from a partnership is $95,000. Her total taxable income from all sources before taking the 20 percent deduction for QBI is $102,000. What is the amount of the Section 199A qualified business income deduction that Grace can take this year? $7,000 $16,600 $19,000 $20,400

The correct answer is (C). The deduction is the lesser of 20 percent of QBI and 20 percent of taxable income. Twenty percent of her QBI is $19,000. Twenty percent of her taxable income is $20,400. Her deduction is $19,000.

Bob would like to invest in bonds and is considering either a taxable bond with an interest rate of 7 percent or a tax-exempt municipal bond of comparable risk and quality with an interest rate of 5 percent. Bob's marginal tax rate is 24 percent. In order to help Bob compare these two bonds, compute the equivalent tax-free rate for the taxable bond. 5.25% 5.00% 5.32% 3.80%

The correct answer is (C). The equivalent tax-free rate for the taxable bond is 5.32 percent. 0.07 x (1 - 0.24) = 5.32%

Last year, Alessandra bought a home in Detroit. Alessandra is now considering selling her home and buying a new home, but she is not sure whether she can qualify for a prorated exclusion of the gain on the sale of her Detroit home under Section 121 of the IRC. Alessandra would qualify for a prorated exclusion under all of the following circumstances EXCEPT: Allessandra has decided to sell her house because she has accepted a new job in New York City. Her last job was in Detroit. Allessandra has decided to sell her house because her personal physician recommended that she move to the desert in Arizona because the air pollution in Detroit was causing her asthma to get worse. Allessandra has decided to sell her house because her dog, Tinkerbell, has arthritis and cannot walk down the stairs in Allessandra's home. Allessandra has decided to sell her house because she is bothered by excessive noise from a nearby airport.

The correct answer is (C). The health of a pet is not considered a change in health that justifies a partial exclusion under Section 121. All of the other options are reasons that would justify a partial exclusion under Section 121.

Which of the following statements concerning the tax deductibility of executive compensation is correct? Publicly traded companies may not deduct compensation payments made to any employee to the extent that the employee's compensation exceeds $1,000,000. The compensation cap applies to all compensation (cash and noncash) received by the executive in the taxable year. Contributions to qualified retirement plans and benefits that would not otherwise be taxable are not considered to be compensation subject to the $1,000,000 cap. Private companies may elect to apply the deductibility cap for compensation to only the top five officers of the company.

The correct answer is (C). The limitation on the deductibility of executive compensation only applies to the CEO, the CFO, and the three most highly compensated executives. It does not apply to private (nonpublicly traded) companies. When applying the $1,000,000 deduction cap, contributions to qualified retirement plans are not considered to be part of the executive's compensation.

Andrei, aged 45, is an active participant in his employer's defined-benefit retirement plan, but he would also like to make a deductible contribution to a traditional IRA this year. Andrei is married, files a joint return with his wife, and has an adjusted gross income (AGI) of $112,000 in 2020. What is the maximum deductible contribution that Andrei can make to a traditional IRA? $2,400 $3,000 $3,600 $6,000

The correct answer is (C). The phase-out range for taxpayers who are active participants and use the married-filing-jointly filing status is $104,000 to $124,000 for 2020. Since Andrei's AGI is within this range, he may not make a full $6,000 deductible contribution to a traditional IRA, but he may make a reduced deductible contribution, as calculated by the following formula:reduction = contribution limit × [(AGI − lower limit) ÷ $20,000)]Therefore Andrei's deductible contribution is reduced by $2,400, or $6,000 × [($112,000 − $104,000) ÷ $20,000]. The maximum deductible contribution that Andrei can make to a traditional IRA is $3,600, or $6,000 − $2,400.

All the following are disadvantages of the sole proprietorship as a business form EXCEPT: The ability of the sole proprietor to raise capital is usually limited to his or her own resources. The transferability of the business is severely limited. There are many requirements to starting a sole proprietorship that make it difficult to get this type of business up and running. A sole proprietor has unlimited liability for business debts.

The correct answer is (C). There are practically no requirements that make the initiation of a sole proprietorship difficult. However, a sole proprietor commonly has a problem raising large amounts of capital, and the transferability of the business is usually limited because the input of the proprietor himself or herself is the factor that makes the business viable and valuable. On top of all of these issues is the unlimited liability the sole proprietor has for his or her business debts.

Which of the following are tax credits that reduce the tax liability calculated on taxable income? I. Student Loan Interest tax credit II. Child tax credit III. Earned Income credit IV. Retirement Saver's credit I, II, and III II and III II, III, and IV I, II, III, and IV

The correct answer is (C). There is no such thing as a Student Loan Interest tax credit. All of the other items are credits against the calculated tax.

Tony's son, John, completed his junior year of college in 2020. Tony paid $3,000 in qualified expenses for John in 2020. Tony is a married-filing-jointly taxpayer and has an adjusted gross income (AGI) of $150,000 for the current year. What education credit will provide Tony the highest credit, and how much is that credit? Tony can claim a Lifetime Learning credit in the amount of $600. Tony can claim a Lifetime Learning credit in the amount of $3,000. Tony can claim an American Opportunity tax credit in the amount of $2,250. Tony can claim an American Opportunity tax credit in the amount of $2,500.

The correct answer is (C). Tony can claim an American Opportunity tax credit in the amount of $2,250.($2,000 × 100%) + ($1,000 × 25%) = $2,250

All of the following statements concerning health savings accounts (HSAs) are correct EXCEPT: Excess contributions to HSAs are subject to a 6 percent penalty tax. When a self-employed individual makes a contribution to an HSA, the contribution is disregarded when calculating self-employment taxes for Social Security. Unless contributions to HSAs are distributed to cover health care costs by the end of the taxable year, they are forfeited. Distributions from HSAs that are not used to cover medical expenses are subject to a 20 percent penalty until the taxpayer reaches age 65.

The correct answer is (C). Unlike for health care flexible spending accounts (which are accounts maintained by employers that allow employees to allocate part of their income on a pretax basis to the account), amounts contributed to HSAs that are not used by the end of the taxable year can be carried forward and used in future years. Excess contributions to HSAs are subject to a 6 percent penalty tax, and early distributions used for nonqualified expenses (before age 65) are subject to a 20 percent penalty tax. Furthermore, contributions to HSAs are not taken into consideration when determining the amount of income subject to Social Security taxes for self-employed individuals.

After 35 years in business for herself, Delilah retired and closed the doors of her office. She gave her desk to her niece, Ruth, who recently completed her degree in a similar field and is opening up her practice. Delilah originally paid $12,000 for the desk, and it was fully depreciated by the time she gave it to Ruth. Ruth used the desk for 2 years and then sold it for $6,000 when she decided to redecorate her office. How will Ruth treat the proceeds from the sale of the gift for income tax purposes? Since Ruth received the desk as a gift, there is no need to pay taxes on the proceeds from the sale. Since the desk was given to Ruth when it was fully depreciated, it is "loss property," and the $6,000 proceeds will not be taxable because it fell between Ruth's gain basis and loss basis in the transaction. Ruth will recognize $6,000 of ordinary income on the sale. Ruth will recognize $6,000 of long-term capital gain on the sale.

The correct answer is (C). When a gift of Section 1231 property is made, the depreciation recapture potential, as well as the taxpayer's basis, is carried over to the new owner. Ruth received the desk with a basis of $0 and a potential for up to $12,000 of depreciation recapture. Since Ruth sold the desk for $6,000, the entire sales proceeds will constitute depreciation recapture and will be taxed at ordinary income tax rates.

During the current year, a taxpayer collected $600 interest on U.S. Treasury bills, $900 interest on a local municipal bond, and $200 from a state income tax refund. (She itemized her deductions last year.) She also received $400 in dividends from a U.S. common stock. Her gross income from the above is $400. $1,000. $1,200. $2,100.

The correct answer is (C). 1,200 Interest on the municipal bond is excluded under Section 103. Her gross income is $1,200, or $600 + $200 + $400. If the taxpayer had taken the standard deduction last year, she would not have included the state income tax refund since no benefit had been received.

Emilia owns an event planning company that specializes in very high-end events. Several years ago, Emilia purchased a magnificent chocolate fountain for $3,000 and has since taken $1,200 in depreciation deductions on the fountain. Emilia is now ready to replace the fountain with tools for creating ice sculptures, but she does not know how to calculate the tax consequences of selling the fountain. Which of the following statements is true regarding the tax consequences of selling the fountain? If Emilia sells the chocolate fountain for $1,800, she will have a $1,200 ordinary loss. If Emilia sells the chocolate fountain for $1,700, she will have a $100 capital loss. If Emilia sells the chocolate fountain for $2,000, she will have an ordinary gain of $200 and no capital gain. If Emilia sells the chocolate fountain for $3,300, she will have a $1,500 capital gain.

The correct answer is (C). If Emilia sells the chocolate fountain for $2,000, she will have an ordinary gain of $200 and no capital gain. Amount Realized: Option (A)- $1,800 Option (B)- $1,700 Option (C)- $2,000 Option (D)- $3,300 Adjusted Basis: Option A:$1,800 Option B: $1,800 Option C: $1,800 Option D: $1,800 Gain/Loss: Option A: $0 Option B: -$100 Option C: $200 Option D: $1,500 Tax Impact: Option A: None Option B: Ordinary loss Option C: Ordinary gain Option D: Part ordinary gain ($1,200)/ Part capital gain ($300) Economic Reality: Option A: Depreciation estimate was perfect Option B: Taxpayer took too little depreciation Option C: Taxpayer took too much depreciation Option D: Taxpayer took too little depreciation and asset appreciated

Last year, Jackson DeSoto sold securities from his portfolio and realized the following gains and losses: • $14,000 net long-term capital loss • $600 net short-term capital gain • $400 net short-term capital loss Which of the following amounts is the carryover that Jackson can take to future years? $7,000 $7,800 $10,800 $14,400

The correct answer is (C).Jackson has a net long-term loss of $14,000 and a net short-term gain of $200, resulting in a net long-term loss of $13,800. Of this net loss, $3,000 can be deducted from gross income, leaving an unused long-term loss of $10,800 as a carryover to future years.

Which of the following activities is least likely to be classified as a hobby? The collection and restoration of classic cars that are not held for resale The collection and restoration of guitars that are not held for resale The raising of horses on a privately owned farm The opening of a brewery in an industrial park

The correct answer is (D). The collection and restoration of classic cars and guitars are activities likely to be engaged in for personal pleasure since these items are not being held for resale in an attempt to make a profit. The raising of horses is commonly classified as a hobby. While the opening of the brewery might bring personal pleasure to the owner(s), breweries are businesses, and the opening of the brewery in an industrial park signals that this is an activity engaged in for the pursuit of profit, as opposed to brewing beer at home for personal consumption.

Cameron, an unmarried taxpayer using the single filing status, received $16,000 of Social Security retirement benefits this year. Cameron also received $5,000 of interest income and $45,000 of income from her retirement plan during the year. How much of Cameron's Social Security benefits must be included in her gross income this year? $0 $8,000 $13,600 $16,000

The correct answer is (C).Since her modified adjusted gross income (MAGI) of $50,000 plus one-half of her Social Security benefits (0.5 × $16,000 = $8,000) exceeds her adjusted base amount of $34,000, she must calculate her includible Social Security benefits using the lesser of the amount calculated using formula 3 or 4. Formula 3: 0.85 × $16,000 = $13,600Formula 4: 0.85 × [$50,000 + (0.50 × $16,000) − $34,000] = $20,400 plus the least of $4,500, the amount calculated using formula 1, and the amount calculated using formula 2Formula 1: 0.50 × $16,000 = $8,000Formula 2: 0.50 × [$50,000 + (0.50 × $16,000) − $25,000] = $16,500 $4,500 is the least of these three numbers. The formula 4 total is $24,900, or $20,400 + $4,500. The lesser of the amounts for formula 3 and 4 is $13,600. Therefore $13,600 of the Social Security benefits must be included in Cameron's gross income.

When an employee of the ABC Company elects salary reduction for a flexible spending account (FSA), which of the following statements is correct? Under the FSA, dependent care benefits can be provided for up to two children. The FSA is typically funded by employee and employer contributions. The salary reduction is subject to Social Security taxes but not to income taxes. If the employee does not use all of the health care salary reduction, the balance is forfeited.

The correct answer is (D) If the employee does not use all of the health care salary reduction, the balance is forfeited. (Note: While plans that do not allow an extension through March of the following year can now be amended to allow up to $500 from a health FSA to carry over, the general rule is still "use it or lose it.") Dependent care benefits are limited by dollar amount, not by number of children. The FSA is typically funded only by employee contributions. The salary reduction is not subject to Social Security or income taxes.

Zihan recently purchased a new machine for his business. The price of the machine was $12,000, but Zihan also paid $100 in sales tax, $300 in freight, and $1,000 in installation and testing costs. What is Zihan's basis in the new machine? $12,100 $12,400 $13,000 $13,400

The correct answer is (D). All costs to get the asset into operation are included in the cost basis of the asset. Therefore Zihan's cost basis includes the price of the machine and the costs for sales tax, freight, and installation and testing

Which of the following is a risk faced by the owner of a business entity? The death or disability of another owner The retirement of another owner Bankruptcy All of the above

The correct answer is (D). All of the options are potential risks faced by the owner of a business entity.

In which of the following entities will all persons who participate in management be protected from liability beyond their investment? I. A C corporation II. An S corporation III. A limited liability company IV. A limited partnership I only I and IV II and III I, II, and III

The correct answer is (D). All shareholders and managers will be protected by limited liability in a C corporation and in an S corporation. All managers who are members of a limited liability company are also protected by limited liability. With a limited partnership, the general partner is the manager and has unlimited liability.

After 1989, Megan purchased series EE savings bonds for $4,000 at the age of 26. This year she redeemed the bonds for $8,000 and paid qualified higher education expenses in the amount of $5,000 for her daughter. How much interest can Megan exclude from her gross income this year? $0 $4,000 $1,500 $2,500

The correct answer is (D). Because Megan did not use all of the proceeds from the bond redemption to pay for qualified education expenses, she will be required to include part of the interest income from the bonds in her gross income. Megan may exclude $2,500 of interest income from her gross income; that is, ($5,000 ÷ $8,000) x $4,000. Therefore Megan must include $1,500 in her gross income, or $4,000 - $2,500.

LMN, Inc., has a net income of $650,000 before depreciation and has purchased one used piece of equipment during the year for $1,200,000. Which of the following depreciation strategies will allow LMN to claim the largest depreciation deduction in the current year? The use of Section 179 The use of the modified accelerated cost recovery system (MACRS) The use of Section 179 and MACRS The use of bonus depreciation

The correct answer is (D). Bonus depreciation allows a 100 percent write-off against income, whereas Section 179 is limited to the lesser of $1,040,000 (for 2020) and the net income of $650,000. While MACRS allows depreciation to be claimed on an accelerated basis, it will take 5 years to recover the full cost of the equipment according to the depreciation tables.

Corinne is an unmarried taxpayer with three children at the ages of 2, 4, and 6. Corinne's modified adjusted gross income (MAGI) is $85,000. What is the amount of the child tax credit that Corinne can claim? $0 $1,000 $2,500 $6,000

The correct answer is (D). Corinne has three children under the age of 17. The child tax credit is $2,000 per child. While the allowable tax credit amount is reduced by $50 for each $1,000 that a single-filing taxpayer's modified adjusted gross income exceeds $200,000 (TCJA 2017), Corinne's MAGI does not exceed the threshold. Therefore Corinne can claim a total of $6,000 under the Child tax credit, or $2,000 × 3.

Doug and Katie are trying to calculate their gross income for the current year. Which of the following items should they include in their gross income? I. Dividends earned on a stock owned by Doug that were reinvested II. Money borrowed by Doug and Katie from Chase Bank III. Gain from the sale of Doug's classic guitar collection IV. Interest earned on a loan made by Katie to her sister I and II III and IV I, II, and III I, III, and IV

The correct answer is (D). Dividends received, even if reinvested, are included in income, and therefore Option I is correct. Borrowed money is excluded from gross income, and therefore option II is incorrect. Gain on the sale of assets (Option III) and interest income (Option IV) are both included in gross income.

Helen is covered by a group term life insurance policy that provides coverage equal to three times her annual salary of $125,000. Helen's employer pays the entire cost of the policy, for which the uniform premium is $0.15 per $1,000 of coverage per month. How much of this premium is taxable to Helen? $1,500 $1,000 $48.75 $585

The correct answer is (D). The cost of up to $50,000 of coverage is nontaxable.$375,000 − $50,000 = $325,000[($325,000 ÷ $1,000) × $0.15] × 12 months = $585 taxable coverage

On January 1, 2018, Axel and Sasha divorced. Under the terms of the divorce decree, Sasha was given custody of their only child, Marcy, who is 16 years old. The court decree also requires Axel to pay Sasha $1,000 a month in alimony for 2 years (that is, until Marcy turns 18), followed by $800 a month for the 4 years after that. Which of the following statements concerning the payments that Axel makes to Sasha is correct? Axel will be able to reduce his adjusted gross income (AGI) by the $12,000 in alimony payments he makes to Sasha this year. Sasha will not be required to report any of the payments that Axel makes to her as income on her tax return. Axel will not be able to take a deduction for the alimony payments made due to the alimony recapture rule. Axel's tax deduction will be limited to $9,600 per year during the 6-year period.

The correct answer is (D). For income tax purposes, alimony payments that are reduced when a minor child reaches the age of majority (18 in most states) are considered to be a form of child support. Since Axel's alimony payments will be reduced by $200 per month once Marcy reaches age 18, $200 of the payment will be considered child support, leaving the remaining $800 as alimony. As a result, Axel will only be able to deduct from his income, and Sasha will be required to include in her income, $9,600 per year.

Henry, single and aged 42, was divorced after 12/31/2018. He had the following items of income and expense for the current tax year. Wages: $60,000 Interest: $1,000 Inheritance: $50,000 Alimony paid: $10,000 Child support paid: $8,000 Federal taxes paid: $5,000 State income taxes paid: $2,000 Medical expenses: $7,500 What is Henry's adjusted gross income (AGI)? $49,000 $51,000 $59,000 $61,000

The correct answer is (D). His income includes the wages and interest. The alimony paid is not deductible for AGI if the divorce decree is post 12/31/2018. Therefore his AGI is $61,000.$60,000 + $1,000 = $61,000

All of the following statements regarding the deduction of costs associated with investigating the purchase of a new line of business are correct EXCEPT: If the new line of business is not purchased, no deduction is permissible. If the new line of business is purchased and it is in the same line of business as the current trade or business operation, the cost of investigating the new business is fully deductible. The ability to deduct the cost of investigating a new line of business is often overlooked by taxpayers. If the new line of business is purchased and it is in a different line of business than the current trade or business operation, the costs of investigation cannot be deducted.

The correct answer is (D). If the new line of business is purchased and it is in a different line of business than the current trade or business operation, the costs of investigation are subject to capitalization rules as a new line of business. In the case of a new (and unrelated) line of business, up to $5,000 of start-up costs (reduced by the amount that start-up expenditures exceed $50,000 but not to the point where the start-up costs fall below $0) can be deducted during the first year of operations.

Jennifer, who is single, is a partner in a law firm with one other individual. Jennifer's share of partnership income is $75,000 and her total taxable income (excluding the qualified business income deduction) is $285,000, placing her in the 35 percent marginal tax bracket. What is Jennifer's adjusted marginal tax rate on the partnership income after factoring in the Section 199A qualified business income (QBI) deduction? 7% 24% 28% 35%

The correct answer is (D). Jennifer is in the 35 percent bracket, and her income is above the phaseout range for a specified service trade or business (SSTB); therefore she will not be eligible for the QBI deduction.

Jen and Carlos purchased a vacation home 12 years ago for $250,000. Now, the couple is getting divorced. As part of the divorce settlement, Jen receives the vacation home, which is now worth $1,000,000. Which of the following statements is true? If Jen sells the vacation home 6 months after receiving it in the divorce settlement, any gain or loss that she has will be short-term. If Jen sells the vacation home for $800,000, she will have a $200,000 loss. In any future sale of the vacation home, Jen and Carlos will each have a basis of $125,000. If Jen sells the vacation home for $1,100,000, she will have a gain of $850,000.

The correct answer is (D). Option (A) is incorrect because Jen has a carryover holding period, so if she sells the vacation home 6 months after receiving it in the divorce settlement, any gain or loss that she has will be long-term. Option (B) is incorrect because Jen has a carryover basis in the property. Therefore her basis is $250,000. Option (C) is incorrect because Carlos no longer has an interest in the property.

Precision Turbo Mechanics (PTM), Inc., purchased a new machine for cleaning and retooling the turbo blades on trucks. The machine cost $30,000, and there was a 10 percent sales tax and a $1,000 delivery and setup fee. What is PTM's basis in the new machine? $30,000 $31,000 $33,000 $34,000

The correct answer is (D). PTM's basis equals the machine cost of $30,000 plus the sales tax of $3,000 and the $1,000 freight.

Which of the following is true regarding real estate activities? Real estate activities are always passive. An individual investor in rental real estate can always consider her real estate activities as active businesses. Closely held C corporations that participate in real estate activities will always be considered active businesses. Real estate professionals may be allowed to consider their real estate activities as active in some circumstances.

The correct answer is (D). Real estate activities are generally passive, but exceptions do apply. An individual investor in rental real estate will generally be subject to the passive-income rules; however, some exceptions apply, such as the real estate professional exception that applies to taxpayers who spend more than half of their time during the year materially participating in real estate activities and meet the 750-hour requirement. Closely held C corporations are also eligible if more than 50 percent of the gross receipts of the corporation are derived from real property trades or businesses in which the corporation materially participates.

All of the following statements regarding depreciation recapture are correct EXCEPT: Applying depreciation recapture is not optional. Depreciation recapture applies to Section 1231 assets under both Section 1245 and Section 1250. Depreciation recapture exists to offset the benefit the taxpayer previously received from claiming deductions that reduced his or her ordinary income. Depreciation recapture only applies to assets sold within 5 years of their original purchase.

The correct answer is (D). Statements (A) through (C) are all correct. Statement (D) is incorrect because depreciation recapture applies to all assets regardless of when the asset is sold and regardless of whether or not depreciation was previously claimed.

Bill and Jen Walters purchased their home in 2007 and took out a mortgage of $1,000,000. The current loan balance is $875,000, and the fair market value of the home is $1,400,000. The rate on their existing loan is 6 percent, and the current rate for home loans is 4.5 percent, so the Walters would like to refinance to take advantage of the lower interest rate. If the Walters refinance the full $875,000 balance on their mortgage on January 1 of this year, what amount of interest is deductible as an itemized deduction this year? $0 $33,750 $34,453 $39,375

The correct answer is (D). Their original mortgage was taken out before December 15, 2017, allowing them to retain the right to deduct the interest on up to $1,000,000 of debt so long as they do not refinance more than the existing acquisition loan balance.$875,000 × 4.5% = $39,375

Bethann and Skyler, who are married filing jointly, have three children (aged 5, 9, and 17) who are dependents. The couple has a W-2 income of $90,000. They have $13,500 in taxes withheld. Bethann and Skyler have the following expenses for the year: Interest on a home equity loan issued for $100,000 and used for secondary education: $2,500 Interest on their first mortgage with an original loan balance of $450,000: $12,000 Property taxes on a personal residence: $8,000 State income tax: $8,000 State sales tax: $4,500 What is Bethann and Skyler's tax refund or tax due after all deductions and credits? (Assume a flat tax rate of 12 percent and that the tax year is 2020.) $2,984 tax due $7,824 refund due $10,016 refund due $10,176 refund due

The correct answer is (D). Their refund can be determined by subtracting the applicable credits and their withholdings from their tax liability. They are eligible for a Child tax credit for the 5-year-old and the 9-year-old and a Family tax credit for the 17-year-old. $90,000 of income minus the standard deduction of $24,800 results in $65,200. Taxed at a rate of 12 percent, their tax liability is $7,824 0.12 × $65,200 = $7,824 Child credit: 2 × $2,000 = $4,000 Family credit: $500 Taxes withheld: $13,500 $7,824 − $4,000 − $500 − $13,500 = -$10,176 Bethann and Skyler are owed a $10,176 refund.

Vinnie owns a life insurance contract. Vinnie recently reassessed his insurance needs and decided that he would like to exchange his current life insurance contract for a different insurance product. Which of the following transactions might result in gain realization and recognition? Vinnie trades his life insurance contract for a different life insurance policy. Vinnie trades his life insurance contract for a modified endowment contract. Vinnie trades his life insurance contract for an annuity. None of the above transactions would result in the realization and recognition of gain.

The correct answer is (D). Vinnie can trade his life insurance contract for another life insurance contract, a modified endowment contract, or an annuity and be eligible for the deferral of gain under Section 1035 of the Internal Revenue Code.

Trevor died 2 months after he purchased Sunnydale for $30,000. The fair market value of Sunnydale as of the date of Trevor's death was $32,000. He left Sunnydale to his son, Kirk. Since Kirk was the only beneficiary of the estate and there were no estate taxes due, the title to the property was transferred to Kirk within one month of Trevor's death. Two weeks after receiving title to the property, Kirk sold Sunnydale for $35,000. What is the amount and type of income that Kirk will report on the sale? A $5,000 short-term capital gain A $5,000 long-term capital gain A $3,000 short-term capital gain A $3,000 long-term capital gain

The correct answer is (D). When Trevor died, the basis of Sunnydale qualified for a step to fair market value under IRC Section 1014. Kirk's basis in the property is therefore $32,000. Since he sold the property for $35,000 and his basis was $32,000, Kirk's gain is $3,000. Even though Trevor purchased the property 3.5 months before it was sold, Kirk's gain will be a long-term capital gain. Any property received through the estate of a decedent automatically qualifies for long-term capital-gains treatment.

Toni owns a downtown office building. Toni originally purchased the building for $1,500,000 and took straight-line depreciation deductions of $800,000. What will be the tax consequences if Toni sells the building for $1,600,000? (Ignore the Medicare tax on net investment income.) Toni will have ordinary income of $0. Toni will have $800,000 of gain taxed at 25 percent. Toni will have a Section 1231 gain of $100,000 taxed at preferential long-term capital-gains tax rates. All of the above will occur.

The correct answer is (D). Sale Price: $1,600,000 Less Adjusted Basis (that is, $1,500,000 − $800,000): −$700,000 Equals Gain: $900,000 Breakdown of Gain: • Toni will have a $800,000 unrecaptured Section 1250 gain (straight-line depreciation). This is net capital gain taxed at regular rates of up to 25 percent. • Toni will have a $100,000 net capital gain taxed at long-term capital gains tax rates.

Charlotte, aged 70, is single and an employee of Interstellar Corporation. Her only sources of income this year are $80,000 of W-2 wages, $6,000 in Social Security benefits, and $1,000 of interest on State of Georgia bonds. Based on the above, Charlotte's adjusted gross income (AGI) for the current year is $80,000. $81,000. $83,000. $85,100.

The correct answer is (D). Charlotte's adjusted gross income can be calculated as follows: W2 Income: $80,000 Interest on State of Georgia Bonds (tax exempt): $0 Social Security Benefits (85% x $6,000): +$5,100 = $85,100 Her Social Security benefits are 85 percent taxable because of her modified AGI.

This year, Serenity's aggregate income from her businesses was $180,000. Serenity's aggregate business deductions from those businesses were $320,000. Serenity's spouse, Tristan, earns $80,000 of W-2 income. The couple also has $20,000 of investment income. What amount of business loss may be used to offset the income from Tristan's earnings and the investment income? $0 $20,000 $80,000 $100,000

The correct answer is (D).The business deductions total $320,000 and the business income is $180,000 for a total business loss of $140,000. This is less than the $518,000 (in 2020) maximum threshold for married taxpayers filing jointly, so Tristan's income ($80,000) and the investment income ($20,000) can be fully offset by the business loss.

Sirius works for Carpenter Chemical Company, Inc., as an engineer. He has been employed there for the past 5 years and earns a salary of $100,000. Earlier this year, acting on advice given to him by his financial planner, Sirius purchased a qualified long-term care insurance policy and paid a premium of $800. All of his other medical expenses are covered by his employer-sponsored health insurance plan. Which of the following statements concerning the long-term care policy is correct? Since it is treated as a disability policy for income tax purposes, Sirius is not permitted to deduct the cost of long-term care insurance that he purchased. Sirius can deduct the cost of the long-term care insurance as an adjustment to income. Sirius can receive a tax benefit by claiming the cost of the long-term care insurance as a medical expense deduction. Carpenter Chemical Company could have provided the long-term care policy to Sirius as an employee benefit even if it did not provide similar coverage for other employees.

The correct answer is (D.) Qualified long-term care insurance contracts are treated as health insurance contracts for income tax purposes and are deductible. Sirius cannot deduct the cost as an adjustment to income because he is an employee; only self-employed individuals can receive above-the-line deductions for long-term care premiums that are personally paid by them. Companies can, however, provide long-term care coverage for their employees on a discriminatory basis. There is no need to cover all employees; the employer can pick and choose those employees whom he or she wishes to cover, so option (D) is correct. Even though Sirius can claim the cost as a medical expense deduction on his Itemized Deduction schedule, he will not get any tax benefit because medical expenses are only deductible to the extent that they exceed 10 percent of a person's AGI.

Kelly, aged 14 and a dependent, has $2,600 in interest and dividends and $12,750 in earned income from a part-time job. What is Kelly's standard deduction in 2020? $2,200 $13,100 $12,400 $12,750

The standard deduction for a single person is limited to $12,400.


Set pelajaran terkait

Medical Sociology Exam 2 Ch 8-16

View Set

3. Understanding Income Statements

View Set

Chapter 17: Infants, Children, and Adolescents

View Set